Phép phủ với hệ các hình trịn bằng nhau

Một phần của tài liệu (LUẬN văn THẠC sĩ) hình học tổ hợp với các phương pháp chứng minh luận văn ths toán học 84601 (Trang 66)

3.3 Phép phủ và đóng gói

3.3.2 Phép phủ với hệ các hình trịn bằng nhau

Giả sử S = {D1,D2, . . . ,Dn}, với n ≥ 1, là một hệ các hình trịn với bán kính bằng R, vàM0là đối tượng phẳng bị chặn. Mục đích trong các bài tốn sau là tìm, với ncố định, R nhỏ nhất sao cho tậpM0 cho trước có thể được phủ bởi hệ S, hoặc để quyết định giá trị cho trước R liệu có phủ tất cả. Bài tốn có thể đáng quan tâm ngay cả trong trường hợp n = 1 nếu ta khơng có đủ thơng tin về đối tượng M0 (ví dụ trong Bài 3.3.5 ta biết độ dài của chu vi một đa giácM0). Một kết quả quan trọng trong hướng này là định lý của Jung:

Nếu khoảng cách giữa hai điểm bất kỳ của hình phẳngM0 lớn nhất bằng 1,

thìM0 có thể được phủ bởi một hình trịn với bán kính √1 3.

Trên thực tế, nếu n ≥ 2, thì phép phủ M0 bởi hệ S các hình trịn bằng nhau có nghĩa là ta có thể chia hệ M0 thành nhiều nhất n phần rời nhauM1,M2, . . . ,Mncó tính chất chung sau: khoảng cách giữa hai

điểm “xa nhất” Xi,Yi ∈ Mi với mọi i = 1, 2, . . . ,n không vượt quá 2R. (Lưu ý rằng điều kiện này không là đủ đối với sự tồn tại của một phủ như vậy.)

Bài 3.3.4. Cho M là một tập hợp các điểm bất kỳ (vô hạn hoặc hữu hạn) trên

mặt phẳng với tính chất mà mọi bộ ba điểm từMcó thể được phủ bởi một hình trịn có bán kính R. Chứng minh rằng tất cả các điểm của tập M có thể được phủ bởi một hình trịn có bán kính R.

Giải. Ta xét hệ S các hình trịn D(X,R) bán kính R và tâm X, trong đó X ∈ M. Vì ta có thể phủ bất kỳ bộ ba điểm từM với một hình trịn bán kính R, điều này có nghĩa là ba hình trịn bất kỳ trong S có một điểm chung. Theo định lý Helly điều này kéo theo tất cả các hình trịn trong

S có một điểm chung A. Nhưng khi đó hình trịn D(A,R) phủ tồn bộ

tậpM.

Bài 3.3.5. Chứng minh rằng đa giác M bất kỳ với chu vi 2a có thể được phủ bởi một hình trịn có bán kính R = 2a. Chứng minh rằng nếu 0 < d < a, thì

có một đa giác Mvới chu vi 2a không thể được phủ bởi một hình trịn với bán

kính d2.

Giải. Trên biên của đa giác M ta chọn các điểm A,B sao cho theo cả hai hướng dọc biên, chúng đều có khoảng cách a từ mỗi điểm. Do đó

|AB| < a, và với mọi điểm M trên biên của M ta có |AM|+ |BM| ≤

a. Ký hiệu S là trung điểm của đoạn thẳng AB, và bây giờ ta chứng minh rằng hình trịn D(S,2a) phủ tồn bộ biên của M (và do đó phủ tồn bộ đa giác M). Để làm điều này, ta chọn một điểm M bất kỳ trên biên của M, và dựng điểm M0 đối xứng với M qua trung điểm S(hình 3.26). Theo bất đẳng thức tam giác, |MM0| ≤ |MA|+|MA0|, tức là, |

SM ≤ 12(|MA|+|AM0|). Vì |AM0| = |BM| và |AM|+|BM| ≤ a, ta có

Hình 3.26

Một ví dụ về đa giác thỏa mãn các điều kiện phần hai của bài toán là một tam giác cân với đáy(a−d)và các cạnh bên bằng 12(a+d). Rõ ràng là một tam giác tồn tại với mọi0 < d < a, và có tính chất mong muốn, vì 12(a+d) > 12(d+d) = d; có nghĩa là cạnh bên của tam giác khơng thể

được phủ bởi một hình trịn bán kính d2.

Bài 3.3.6. Chứng minh rằng một hình trịn bán kínhRkhơng thể được phủ bởi hai hình trịn với bán kính nhỏ hơn R.

Hình 3.27

Giải. Ta chứng minh điều này bằng phản chứng. Giả sử rằng hình trịn

D(S,R)) được phủ bởi hai hình trịnD1(S1,r1),D2(S2,r2), trong đómax{r1,r2} <

R. Đầu tiên giả sử rằng S1 = S (hình 3.27a). Thì rõ ràng các điểm đầu mút X,Y trên đường kính của D khơng nằm trong D1, và do đó

X,Y ∈ D2; có nghĩa là 2r2 ≥ |X,Y| ∈ 2Rmâu thuẫn. Trong trường hợp S1 6= Sta có |S1X| > R > r1 (xem hình 3.27b); do đóX ∈ D/ 1 vàX ∈ D2, và tương tự ta có đượcY ∈ D2. Vì vậy, nói chung, X,Y ∈ D2 khơng thể

xảy ra, vì|XY| = 2R > 2r2.

3.3.3 Bài tốn về đóng gói

Dưới đây ta trình bày một số bài tốn về tính chất của hệ S của các đối tượng hình học được đóng gói trong một đa giácM.

Bài 3.3.7. Trong một hình chữ nhật có kích thước 20×25 ta có 120 ơ vng

đơn vị. Chứng minh rằng ta cũng có thể đặt một hình trịn D với đường kính bằng 1 trong hình chữ nhật sao cho nó khơng phủ lên bất kỳ ơ vng nào cho trước.

Hình 3.28

Giải.Như hình 3.28, ta “co” hình chữ nhậtR đã cho bằng một khung có chiều rộng x = 12, và ta “mở rộng” mỗi một trong 120 ô vuông bằng tập tất cả những điểm có khoảng cách tối đa bằng 12 từ hình vng. Mỗi ơ vng “mở rộng” có diện tích S = 1 +4·1· 12 +π(12)2 = 3+ π

4, và do đó tổng S0 diện tích của tất cả các ơ vng “mở rộng” là 120·(3+ π

4); đồng thời, diện tích của hình chữ nhật “co” lại là19·24 = 456. Theo bất đẳng thức120·(3+ π

4) < 456 ta thấy rằng một số điểmX của hình chữ nhật “co” khơng nằm trong bất kỳ hình vng “mở rộng” nào. Do đó

Bài 3.3.8. Trên một bàn trịn với bán kính R n > 1 khơng phủ nhau các đồng xu bằng nhau có bán kính rsao cho không thể đặt thêm đồng xu như vậy trên bàn mà không phủ lên một vài trong số n đồng xu. Chứng minh rằng ta có bất đẳng thức 1 2 R r −1 < √ n < R r . (3.15)

Giải. Vì các đồng xu khơng phủ lên nhau, tổng diện tích của chúng nhỏ hơn so với diện tích của bàn; do đó ta có n·πr2 < πR2, suy ra vế phải của bất đẳng thức (3.15). Còn lại, chứng minh vế trái. Ký hiệu các đồng xu đã cho là Di(Si,r),i ∈ I = {1, 2, . . . ,n}, và bàn là D(S,R). Nếu ta tăng bán kính của tất cả các đồng xu từ r đến 2r, ta có được đồng xu mới D∗i (Si, 2r), và nếu ta giảm bán kính của bàn từ R thành R−r, ta có được một “bàn” mớiD∗(S,R−r). Hệ hình trịnD∗i phủ hình trịnD∗, vì nếu khơng sẽ tồn tại một số điểm X ∈ D∗ với tính chất X ∈ D/ i∗ (i ∈ I), tức là, |XS| ≤ R−r và |XSi| > 2r i ∈ I, ta có thể thêm một đồng xu mới Dn+1(X,r) vào các đồng xu ban đầu, với tính chất Dn+1 ⊆ D và

Di ∩ Dn+1 = ∅i ∈ I, mà theo giả thuyết của bài tốn là khơng thể. Do đó, vì n hình trịn Di∗ phủ hình trịn D∗, tổng diện tích của chúng thỏa mãnn·π·(2r)2 ≥ π·(R−r)2, và rõ ràng đẳng thức không xảy ra. Biến đổi dễ dàng thu được bất đẳng thức chặt cho vế trái ở trong (3.15).

Bài 3.3.9. Giả sử rằng một hình trịn D có bán kính n chứa4n đoạn thẳng có độ dài đơn vị, và chứa đường thẳng l cho trước. Chứng minh rằng tồn tại một đường thẳng l0 sao cho hoặc song song hoặc vng góc với l và cắt ít nhất hai trong số các đoạn thẳng đã cho.

Giải. Ta dựng các đoạn thẳng A1A2 và B1B2 là các đường kính của hình trịn D sao cho A1A2 k l và B1B2 ⊥ l. Ta ký hiệu các phép chiếu trực giao của đoạn thẳng đã cho Ui(1 ≤ i ≤ 4n) lên đường A1A2, B1B2 tương ứng bởiVi, Wi. Mỗi bộ Vi,Wi, là một đoạn thẳng (hoặc có thể là một điểm), và độ dài của chúng làai = |Vi|,bi = |Wi| rõ ràng thỏa mãn ai +bi ≥ |Ui| = 1(1 ≤ i ≤ 4n), và do đó

(a1+a2+. . .+a4n) + (b1+b2+. . .+b4n) ≥ 4n.

Đầu tiên ta giả sử rằng a1+a2+. . .+a4n ≥ b1+b2+. . .+b4n; thì

Dựa vào thực tế mỗi bộV1,V2, . . . ,V4n là tập con của đoạn thẳng A1A2 có chiều dài 2n, bất đẳng thức cuối cùng có nghĩa là hai đoạn Vi,Vj

(i 6= j) có một điểm chungC. Do đó, đường vng góc với đường thẳng lvà đi quaCcắt cả hai đoạnUi,Uj. Tương tự, trong trường hợpa1+a2+

. . .+a4n < b1+b2+. . .+b4n có một đường song song thích hợp vớil.

3.4 Phép tơ màu

Trong phần này, ta sẽ nghiên cứu các tình huống mà tập M gồm các điểm trong mặt phẳng bị chia thành nhiều lớp T1,T2, . . . ,Tn. Nếu có n màu khác nhau và tơ màu tất cả các điểm của lớp Ti màu thứ i; do đó ta có được màu của tập M. Trong mục này, hai câu hỏi sau thường xuyên

xuất hiện:

• Với phép tô màuMbằng một số màu tùy ý cho trước, lớpTi có một tính chất hình học được quy định cho trước khơng?

• Với một cách tơ màuM thích hợp với một số lượng màu nhất định, mỗi lớp Ti có một tính chất nhất định hay khơng? Làm thế nào để việc tô màu như vậy thực hiện được?

Trong bài tốn tơ màu, bài toán nổi tiếng nhất là bài toán bốn màu.

3.4.1 Màu của các điểm

Ta bắt đầu với những bài tốn trong đó ban đầu khơng có điều kiện nào về tập hợp các điểm có màu giống nhau.

Bài 3.4.1. Giả sử mỗi điểm trên mặt phẳng được tô màu bằng một trong ba màu. Chứng minh rằng có hai điểm có cùng màu mà khoảng cách giữa chúng bằng 1.

Giải. Đầu tiên, khẳng định của bài tốn là tầm thường nếu khơng có nhiều hơn hai màu được sử dụng cho việc tơ màu các điểm (khi đó chỉ cần ít nhất hai đỉnh của bất kỳ tam giác đều bất kỳ chiều dài cạnh bằng 1 có cùng màu).

Hình 3.29

Ta giả sử rằng có cách tơ màu các điểm của mặt phẳng sao cho đầu mút của bất kỳ đoạn thẳng nào có chiều dài 1 có màu khác nhau. Đầu tiên ta chứng minh rằng hai điểm A,B bất kỳ với khoảng cách √3 có cùng màu. Thực tế, nếu |AB| = √

3, thì các đường tròn c1(A, 1) và c2(B, 1) giao nhau tại các điểm C 6= D, và 4ACD và 4BCD là các tam giác đều có chiều dài cạnh bằng 1 (hình 3.29). Vì giả sử của ta về màu của các điểm trong mỗi bộ ba {A,C,D} và {B,C,D} khác nhau, các điểm A và B có cùng màu. Bây giờ ta chỉ cần xét tam giác XYZ bất kỳ, với |XY| = |XZ| = √

3và|YZ| = 1. Với những gì ta vừa chứng minh,Y có cùng màu vớiX, vàXcó cùng màu vớiZ. Do đóYvàZcó cùng màu, trong khi|YZ| = 1. Đây là mâu thuẫn, khẳng định được chứng minh.

Bài 3.4.2. Giả sử rằng mỗi điểm của mặt phẳng được tô với một trong hai màu. Chứng minh rằng có ba điểm cùng màu tạo thành các đỉnh của một tam giác đều.

Hình 3.30

Giải. Ta giả sử rằng đối với việc tô màu khẳng định là sai, nghĩa là, mọi tam giác đều trong mặt phẳng có hai đỉnh có màu khác nhau. Ta xét hình lục giác ABCDEF đều với tâm S có màu đen. Một trong các đỉnh, ta xét B, của tam giác đều như BDF phải có màu đen. Từ việc xét 4ASB và

4BSC thì các điểm A,C có màu khác (màu trắng). Nhưng do đó điểm E có màu đen (4ACE) và F có màu trắng (4ESF). Xét điểm G là giao điểm của các đường AB và EF (xem hình 3.30). G có màu trắng hoặc đen. Một trong hai hình tam giác đều AFG và BEG sẽ có cả ba đỉnh

cùng màu, mâu thuẫn.

Bài 3.4.3. Giả sử rằng mỗi điểm của mặt phẳng được tô một trong ba màu. Chứng minh rằng có ba điểm cùng màu tạo thành các đỉnh của một tam giác vuông.

Giải. Ta chọn hai điểm A,B có cùng màu (màu a) trên mặt phẳng. Cho hình trịn k có đường kính AB nội tiếp trong hình vng KLMN, như trong hình 3.31 (các điểm tiếp xúc là các đỉnh của hình vng APBQ). Nếu một trong các điểm P và Q có màu a, thì ta đã chứng minh xong (xét4ABPvà4ABQ). Điều này cũng đúng trong trường hợp PvàQcó cùng màu; do đó ta chỉ cần xét việc tơ màu các đỉnh của tam giác vng XYZ nội tiếp trong hình trịnk sao cho

{X,Y,Z} ∩ {A,B,P,Q} = ∅.

Cịn lại phân tích trường hợp các điểm P và Q có màu khác nhau b,c, và trong đó X = A là điểm X duy nhất trên đoạn KN có màu a (nếu khơng 4BAX sẽ là tam giác mong muốn). Đối với màu của điểm K ta phân biệt giữa hai khả năng. Nếu K có màu b, thì chỉ cần hạn chế trong trường hợp mỗi điểmX ∈ KN, X ∈ {A,/ K}, có màu c(nếu không, đủ để lấy 4PKX); chọn một điểm X ∈/ N một cách tùy ý, ta có được một tam giác vng QNX với các đỉnh có màuc. NếuK có màu cvà nếu 4KNQ

khơng có tính chất mong muốn, thì N có màu b (xem hình 3.31), và do đó đủ để xét việc tô màu của các đỉnh của tam giác vngXYZ nội tiếp trong vịng trịnk0 ngoại tiếp hình chữ nhậtKPQN sao cho

{X,Y,Z} ∩ {K,P,Q,N} = ∅.

3.4.2 Tô màu miền

Bài 3.4.4. Chứng minh rằng nếu mặt phẳng bị chia bởi n đường thẳng thành nhiều miền, thì mỗi miền có thể được tô bằng một trong hai màu sao cho hai miền bất kỳ có cùng màu khơng có nhiều hơn một điểm chung.

Giải. Ta chứng minh khẳng định bằng phép quy nạp với n. Với n = 1 rõ ràng ta chỉ cần tô màu khác nhau cho hai nửa mặt phẳng. Ta giả sử rằng khẳng định đúng với mọi n = k ≥ 1, và xét hệ P gồmk+1đường bất kỳ trong mặt phẳng. Ta chọn một đường bất kỳ l ∈ P và sau đó xét một cách tơ màu miền thích hợp mà mặt phẳng bị chia bởi hệ k đường

Hình 3.32

một hoặc một nửa mặt phẳng còn lại được xác định bởi đường l, trong khi phần còn lại bị cắt thành hai miền mới bởi l. Những cặp miền mới này là lý do tại sao cho phép tô màu cho hệP0 phải được chuyển thành tô màu cho hệP. Ta làm như sau: Trong một nửa mặt phẳng bị giới hạn bởil (phần trên của hình 3.32a) ta giữ ngun phép tơ màu, trong khi ta thay đổi màu của mỗi miền trong nửa mặt phẳng cịn lại (hình 3.32b). Do đó, ta thu được cách tơ màu có thể đối với hệ P, vì các cặp miền lân cận cũng có màu khác nhau trong trường hợp có đường biên chung (một đoạn thẳng có chiều dài dương, một đường hoặc một nửa đường)

là một phần của đườngl. Chứng minh hoàn thành.

Bài 3.4.5. Mặt phẳng bị chia thành nhiều miền bởi n đường thẳng cho trước

(n ≥ 3). Giả sử một số miền được tô màu (với một màu) sao cho hai miền bất kỳ được tơ màu có khơng nhiều hơn một điểm biên chung. Chứng minh rằng số miền được tô màu không vượt quá n23+n.

Giải. Ký hiệu p là số miền được tô màu, với p = |S1| +|S2|, trong đó S1, S2 lần lượt là lớp các miền bị chặn, tương ứng không bị chặn, được tô màu. Bất đẳng thức p ≤ n23+n rõ ràng đúng trong trường hợp hệ các đường bao gồmnđường song song, trong trường hợp này ta cóS1 = ∅

và |S2| ≤ n+1, và hơn nữa, bất đẳng thức n+1 ≤ n23+n đúng với mọi n ≥ 3. Trong các trường hợp cịn lại, ta có được cận mong muốn bằng cách cộng các bất đẳng thức

|S1| ≤ n(n−2)

3 và|S2| ≤ n, (3.16)

mà bây giờ ta sẽ chứng minh. Lưu ý rằng mỗi một trongnđường thẳng

Một phần của tài liệu (LUẬN văn THẠC sĩ) hình học tổ hợp với các phương pháp chứng minh luận văn ths toán học 84601 (Trang 66)

Tải bản đầy đủ (PDF)

(90 trang)